Limits ( 1 )

Calculus Level 3

Find the limit of the following expression:

lim n ( ( n + 1 ) ! n + 1 n ! n ) \lim_{n\to\infty} \left(\sqrt[n+1]{(n+1)!} - \sqrt [n]{n!} \right)

Inspiration: New Methods for Calculations of Some Limits

e \sqrt{e} e 2 e^2 e e 1 e \frac{1}{e} e 2 \frac{e}{2}

This section requires Javascript.
You are seeing this because something didn't load right. We suggest you, (a) try refreshing the page, (b) enabling javascript if it is disabled on your browser and, finally, (c) loading the non-javascript version of this page . We're sorry about the hassle.

1 solution

Chew-Seong Cheong
May 14, 2020

L = lim n ( ( n + 1 ) ! n + 1 n ! n ) By Stirling’s formula: n ! 2 n π ( n e ) n = lim n ( ( 2 ( n + 1 ) π ) 1 2 ( n + 1 ) n + 1 e ( 2 n π ) 1 2 n n e ) Note that lim n ( 2 π ) 1 2 ( n + 1 ) = lim n ( 2 π ) 1 2 n = 1 = 1 e lim n ( n ( ( n + 1 ) 1 2 ( n + 1 ) n 1 2 n ) + ( n + 1 ) 1 2 ( n + 1 ) ) By theorem 1 (see reference) = 1 e lim n n ( n + 1 n ) d n 1 2 n d n + exp ( lim n ln ( n + 1 ) 2 ( n + 1 ) 1 ) An / case, L’H o ˆ pital’s rule applies. = 1 e lim n n 1 + 1 2 n ( 1 2 n 2 ln n 2 n 2 ) + exp ( lim n 1 n + 1 2 1 ) Differentiate up and down w.r.t. n = 0 + e 0 1 = 1 e \begin{aligned} L & = \lim_{n \to \infty} \left(\sqrt[n+1]{(n+1)!} - \sqrt[n]{n!} \right) & \small \blue{\text{By Stirling's formula: }n! \sim \sqrt{2n\pi}\left(\frac ne\right)^n} \\ & = \lim_{n \to \infty} \left((2(n+1)\pi)^{\frac 1{2(n+1)}} \cdot \frac {n+1}e - (2n\pi)^{\frac 1{2n}} \cdot \frac ne \right) & \small \blue{\text{Note that }\lim_{n \to \infty} (2\pi)^{\frac 1{2(n+1)}} = \lim_{n \to \infty} (2\pi)^{\frac 1{2n}} = 1} \\ & = \frac 1e \lim_{n \to \infty} \left(\blue {n \left((n+1)^{\frac 1{2(n+1)}} - n^{\frac 1{2n}} \right)} + \red{(n+1)^{\frac 1{2(n+1)}}} \right) & \small \blue{\text{By theorem 1 (see reference)}} \\ & = \frac 1e \lim_{n \to \infty} \blue {n(n+1-n)\frac {d\ n^{\frac 1{2n}}}{dn}} + \exp \left(\red{\lim_{n \to \infty}\frac {\ln(n+1)}{2(n+1)}}-1 \right) & \small \red{\text{An }\infty/\infty \text{ case, L'Hôpital's rule applies.}} \\ & = \frac 1e \blue{\lim_{n \to \infty} n^{1+\frac 1{2n}} \left(\frac 1{2n^2} - \frac {\ln n}{2n^2} \right)} + \exp \left(\red{\lim_{n \to \infty} \frac {\frac 1{n+1}}2} - 1 \right) & \small \red{\text{Differentiate up and down w.r.t. }n} \\ & = \blue 0 + e^{\red 0-1} = \boxed{\frac 1e} \end{aligned}


References:

  • Theorem 1: lim n n ( f ( a n + 1 ) f ( a n ) ) = lim n n ( a n + 1 a n ) f ( a n ) \displaystyle \lim_{n \to \infty} n(f(a_{n+1}) - f(a_n)) = \lim_{n \to \infty} n(a_{n+1} - a_n) f'(a_n) . Source: New Methods For Calculations of Some Limits
  • L'Hôpital's rule

@Chew-Seong Cheong , I see what you changed now. Still a brilliant solution.

Hana Wehbi - 1 year ago

Log in to reply

My previous solution is inconclusive and wasn't brilliant.

Chew-Seong Cheong - 1 year ago

Log in to reply

I didn’t see it before, now l understood it.

Hana Wehbi - 1 year ago

0 pending reports

×

Problem Loading...

Note Loading...

Set Loading...